Вы находитесь на странице: 1из 45

THE OFFICIAL LSAT—INDIA™

PrepTest No. 6
LSAT—India : All You Need Is Reason™

Form K-2LSI055
Actual 2013 LSAT—India

LSAC.org

Law School Admission Council


TABLE OF CONTENTS

• Introduction to the Law School Admission Test—India............................................................................................................1

• Scoring .....................................................................................................................................................................................1

• The Question Types ................................................................................................................................................................1


• Reading Comprehension Questions ..................................................................................................................................1
• Analytical Reasoning Questions .........................................................................................................................................2
• Logical Reasoning Questions .............................................................................................................................................3

• Taking the PrepTest Under Simulated Conditions................................................................................................................4

• Answer Sheet ...............................................................................................................................................................................5

• The PrepTest ................................................................................................................................................................................7

• Answer Key.................................................................................................................................................................................37

The Law School Admission Council (LSAC) is a nonprofit corporation that provides unique, state-of-the-art products and services to
ease the admission process for law schools and their applicants worldwide. Founded in 1947, the Council is best known for administer-
ing the Law School Admission Test (LSAT®), with about 100,000 tests administered annually at testing centers worldwide. Currently,
222 law schools in the United States, Canada, and Australia are members of the Council and benefit from LSAC’s services, which also
include processing of academic credentials for an average of 60,000 law school applicants annually, providing software and informa-
tion for admission offices and applicants, conducting research, promoting diversity in legal education and the legal profession, and
publishing LSAT preparation materials, among many other services. LSAC electronic applications account for nearly all applications to
American Bar Association–approved law schools.

LSAC does not engage in assessing an applicant’s chances for admission to any law school; all admission decisions are made by indi-
vidual law schools. Law School Admission Test—India, LSAT—India : All You Need Is Reason, and Law School Admission Council are
trademarks of the Law School Admission Council, Inc.

©2016 by Law School Admission Council, Inc.

Law School Admission Council


INTRODUCTION TO THE LAW SCHOOL ADMISSION TEST—INDIA

LSAT—INDIA : ALL YOU NEED IS REASON


The LSAT—India is a test of reasoning and reading skills, not a test to see whether you happened to have memorized
the right facts or equations. You can also be assured that each LSAT—India question will have a single answer that is
clearly best. Before you ever see the questions, each is subjected to exacting reviews by at least 10 professionals with
advanced degrees in fields such as logic, English, and linguistics.

The LSAT—India is a test of acquired, high-level reading, the critical-thinking skills measured by the test for each
informal reasoning, and deductive reasoning skills, candidate in comparison to the others in his or her
developed specifically for use by law schools in India. candidate pool.
Although modeled on the Law School Admission Test
(LSAT), it is adapted to the specific needs of Indian THE QUESTION TYPES
legal education.
Critical-thinking skills are key to success in the practice The multiple-choice questions that make up the LSAT—
of law throughout the world. The education of successful India reflect a broad range of academic disciplines and are
lawyers begins with assessing critical-thinking skills during intended to give no advantage to candidates from a
the law school admission process. particular academic background. The LSAT—India does
This PrepTest is a valuable tool for preparing for the not include questions requiring the mastery of any specific
LSAT—India. It is the actual 2013 LSAT—India. It consists discipline or set of facts. For example, it does not test a
of four, 35-minute sections of multiple-choice questions— candidate’s knowledge of history, political theory, or even
one Analytical Reasoning section, one Reading general knowledge. Rather, it is a test of important critical-
Comprehension section, and two Logical Reasoning thinking skills that a student has acquired over his or her
sections. You can use this practice test most effectively by educational lifetime. Thus, the LSAT—India is different
taking it under timed conditions as outlined in “Taking the from other legal-education admission tests used in India. It
PrepTest Under Simulated Conditions” on page 4 and on measures a different set of skills and, even for those
the reverse side of the sample answer sheet. admission tests that do partially address critical thinking,
We wish you great success with the test and your pursuit it measures those skills in different ways.
of a law degree. The four sections of the test contain three different
question types. The following material presents a general
SCORING discussion of the nature of each question type and some
strategies that can be used in answering them.
Credit is given for each question a test taker answers
correctly, and all questions count equally. There is no Reading Comprehension Questions
negative marking or penalty for guessing, so a candidate
should answer each and every question. Test scores are The purpose of LSAT—India Reading Comprehension
reported on a percentile basis, comparing each questions is to measure the ability to read, with
candidate’s performance to that of the others within his understanding and insight, examples of lengthy and
or her candidate group (Five-Year Integrated LL.B. complex materials similar to those commonly encountered
Programme or One or Two-Year LL.M./ Three-Year LL.B. in law school. The Reading Comprehension section of the
Programme). Scores for one candidate group cannot be LSAT—India contains four sets of reading questions, each
compared to those for the other candidate group since set consisting of a selection of reading material followed
they are based on group performance. So, for example, an by five to eight questions. The reading selections generally
undergraduate candidate earning an LSAT—India score of consist of a single reading passage, but in some tests one
82.5 has performed better on the test than 82.5 percent of of the four reading sets will be based on two shorter
the total undergraduate candidate pool. This score does related passages. Sets with two passages are a variant of
not indicate what the candidate’s standing would be within Reading Comprehension called Comparative Reading.
the post-undergraduate candidate pool. Note also that Reading selections for Reading Comprehension questions
this score does not mean that the candidate answered 82.5 are drawn from subjects such as the humanities, the social
percent of the LSAT—India questions correctly. Thus, sciences, the biological and physical sciences, and issues
LSAT—India scores tell law schools the relative strength of related to the law. Reading Comprehension questions

Law School Admission Council


require you to read carefully and accurately, to determine Note transitions from one idea to the next and examine
the relationships among the various parts of the reading the relationships among the different ideas or parts of a
selection, and to draw reasonable inferences from the passage, or between the two passages in Comparative
material in the selection. The questions may ask about the Reading sets. Consider how and why an author makes
following characteristics of a passage or pair of passages: points and draws conclusions. Be sensitive to implications
of what the passages say.
• the main idea or primary purpose; You may find it helpful to mark key parts of passages.
For example, you might underline main ideas or important
• the meaning or purpose of words or phrases used; arguments, and you might circle transitional words—
“although,” “nevertheless,” “correspondingly,” and the
• information explicitly stated; like—that will help you map the structure of a passage.
Moreover, you might note descriptive words that will help
• information or ideas that can be inferred; you identify an author’s attitude toward a particular idea
or person.
• the organization or structure;
Answering the Questions
• the application of information in a passage to a new • Always read all the answer choices before selecting the
context; and best answer. The best answer choice is the one that
most accurately and completely answers the question
• the author’s attitude as it is revealed in the tone of a being posed.
passage or the language used.
• Respond to the specific question being asked. Do not
Suggested Approach pick an answer choice simply because it is a true
Since reading selections are drawn from many different statement. For example, picking a true statement might
disciplines and sources, you should not be discouraged if yield an incorrect answer to a question in which you are
you encounter material with which you are not familiar. It is asked to identify an author’s position on an issue, since
important to remember that questions are to be answered here you are not being asked to evaluate the truth of the
exclusively on the basis of the information provided in the author’s position but only to correctly identify what that
selection. There is no particular knowledge that you are position is.
expected to bring to the test, and you should not make
inferences based on any prior knowledge of a subject that • Answer the questions only on the basis of the information
you may have. You may, however, wish to defer working on a provided in the selection. Your own views, interpretations,
set of questions that seems particularly difficult or unfamiliar or opinions, and those you have heard from others, may
until after you have dealt with sets you find easier. sometimes conflict with those expressed in a reading
Strategies. In preparing for the test, you should selection; however, you are expected to work within the
experiment with different strategies and decide which work context provided by the reading selection. You should not
most effectively for you. These include: expect to agree with everything you encounter in Reading
Comprehension passages.
• reading the selection very closely and then answering
the questions;
Analytical Reasoning Questions
• reading the questions first, reading the selection closely,
and then returning to the questions; or Analytical Reasoning items are designed to measure your
ability to understand a structure of relationships and to draw
• skimming the selection and questions very quickly, logical conclusions about the structure. You are asked to
then rereading the selection closely and answering make deductions from a set of statements, rules, or
the questions. conditions that describe relationships among entities such
as persons, places, things, or events. They simulate the
Remember that your strategy must be effective for you kinds of detailed analyses of relationships that a law student
under timed conditions. must perform in solving legal problems. For example, a
Reading the selection. Whatever strategy you choose, passage might describe four diplomats sitting around a
you should give the passage or pair of passages at least table, following certain rules of protocol as to who can sit
one careful reading before answering the questions. Try to where. You must answer questions about the implications of
distinguish main ideas from supporting ideas, and the given information, for example, who is sitting
opinions or attitudes from factual, objective information. between diplomats X and Y.

Law School Admission Council


The passage used for each group of questions describes eligible to serve on a committee, consider only those
a common relationship such as the following: people named in the passage unless directed otherwise.
When in doubt, read the conditions in their most obvious
• Assignment: Two parents, P and O, and their children, R sense. Remember, however, that the language in the
and S, must go to the dentist on four consecutive days, conditions is intended to be read for precise meaning. It is
designated 1, 2, 3, and 4; essential to pay particular attention to words that describe
or limit relationships, such as “only,” “exactly,” “never,”
• Ordering: X arrived before Y but after Z; “always,” “must be,” “cannot be,” and the like.
The result of this careful reading will be a clear picture of
• Grouping: A manager is trying to form a project team the structure of the relationships involved, including the
from seven staff members—R, S, T, U, V, W, and X. Each kinds of relationships permitted, the participants in the
staff member has a particular strength—writing, relationships, and the range of actions or attributes
planning, or facilitating; allowed by the relationships for these participants.
Questions are independent. Each question should be
• Spatial: A certain country contains six cities and each city considered separately from the other questions in its set;
is connected to at least one other city by a system of no information, except what is given in the original
roads, some of which are one-way. conditions, should be carried over from one question to
another. In some cases, a question will simply ask for
Careful reading and analysis are necessary to determine conclusions to be drawn from the conditions as originally
the exact nature of the relationships involved. Some given. Some questions may, however, add information to
relationships are fixed (e.g., P and R always sit at the same the original conditions or temporarily suspend one of the
table). Other relationships are variable (e.g., Q must be original conditions for the purpose of that question only.
assigned to either table 1 or table 3). Some relationships For example, if Question 1 adds the information “if P is
that are not stated in the conditions are implied by and can sitting at table 2 ...,” this information should NOT be
be deduced from those that are stated (e.g., if one carried over to any other question in the group.
condition about books on a shelf specifies that Book L is to Highlighting the text; using diagrams. Many people
the left of Book Y, and another specifies that Book P is to find it useful to underline key points in the passage and in
the left of Book L, then it can be deduced that Book P is to each question. In addition, it may prove very helpful to
the left of Book Y). draw a diagram to assist you in finding the solution to
No formal training in logic is required to answer these the problem.
questions correctly. Analytical Reasoning questions are In preparing for the test, you may wish to experiment
intended to be answered using the knowledge, skills, and with different types of diagrams. For a scheduling
reasoning ability generally expected of college students problem, a calendar-like diagram may be helpful. For a
and graduates. spatial relationship problem, a simple map can be a
useful device.
Suggested Approach Even though some people find diagrams to be very
Some people may prefer to answer first those questions helpful, other people seldom use them. And among those
about a passage that seem less difficult and then those that who do regularly use diagrams in solving these problems,
seem more difficult. In general, it is best not to start another there is by no means universal agreement on which kind of
passage before finishing one begun earlier, because much diagram is best for which problem or in which cases a
time can be lost in returning to a passage and diagram is most useful. Do not be concerned if a particular
reestablishing familiarity with its relationships. Do not problem in the test seems to be best approached without
assume that because the conditions for a set of questions the use of a diagram.
look long or complicated, the questions based on those
conditions will necessarily be especially difficult. Logical Reasoning Questions
Reading the passage. In reading the conditions, do not
introduce unwarranted assumptions. For instance, in a set Logical Reasoning questions evaluate your ability to
establishing relationships of height and weight among the understand, analyze, criticize, and complete a variety of
members of a team, do not assume that a person who is arguments. The arguments are contained in short
taller than another person must weigh more than that passages taken from a variety of sources, including letters
person. All the information needed to answer each to the editor, speeches, advertisements, newspaper
question is provided in the passage and the question itself. articles and editorials, informal discussions and
The conditions are designed to be as clear as possible; conversations, as well as articles in the humanities, the
do not interpret them as if they were intended to trick you. social sciences, and the natural sciences.
For example, if a question asks how many people could be

Law School Admission Council


Each Logical Reasoning question requires you to read TAKING THE PREPTEST UNDER SIMULATED
and comprehend a short passage, then answer one or CONDITIONS
two questions about it. The questions test a variety of
abilities involved in reasoning logically and thinking One important way to prepare for the LSAT—India is to
critically. These include: simulate the day of the test by taking a practice test under
actual time constraints. Taking a practice test under timed
• recognizing the point or issue of an argument or dispute; conditions helps you to estimate the amount of time you
can afford to spend on each question in a section and to
• detecting the assumptions involved in an argumentation determine the question types on which you may need
or chain of reasoning; additional practice.
Since the LSAT—India is a timed test, it is important to
• drawing reasonable conclusions from given evidence use your allotted time wisely. During the test, you may work
or premises; only on the section designated by the test supervisor. You
cannot devote extra time to a difficult section and make up
• identifying and applying principles; that time on a section you find easier. In pacing yourself,
and checking your answers, you should think of each section
• identifying the method or structure of an argument or of the test as a separate minitest.
chain of reasoning; Be sure that you answer every question on the test.
When you do not know the correct answer to a question,
• detecting reasoning errors and misinterpretations; first eliminate the responses that you know are incorrect,
then make your best guess among the remaining choices.
• determining how additional evidence or argumentation Do not be afraid to guess, as there is no penalty for
affects an argument or conclusion; and incorrect answers.
Please note that in the LSAT—India, some sections may
• identifying explanations and recognizing resolutions of consist of questions with four answer choices, while the
conflicting facts or arguments. other sections consist of questions with five
answer choices.
The questions do not presuppose knowledge of the When you take the practice test that follows, abide by all
terminology of formal logic. For example, you will not be the requirements specified in the directions and keep
expected to know the meaning of specialized terms such as strictly within the specified time limits. Work without a rest
“ad hominem” or “syllogism.” On the other hand, you will period. When taken under conditions as much like actual
be expected to understand and critique the reasoning testing conditions as possible, the practice test
contained in arguments. This requires that you possess an provides very useful preparation for taking the LSAT—India.
understanding of widely used concepts such as argument, Official directions are included in this practice test so
premise, assumption, and conclusion. that you can approximate actual testing conditions as you
practice. To take the test:
Suggested Approach
Read each question carefully. Make sure that you • Set a timer for 35 minutes. Answer all the questions in
understand the meaning of each part of the question. Make SECTION I. Stop working on that section when the 35
sure that you understand the meaning of each answer minutes have elapsed.
choice and the ways in which it may or may not relate to the
question posed. • Repeat, allowing yourself 35 minutes each for sections II,
Do not pick a response simply because it is a true III, and IV.
statement. Although true, it may not answer the question
posed. • An answer key is provided so that you can evaluate your
Answer each question on the basis of the information that performance on the PrepTest.
is given, even if you do not agree with it. Work within the
context provided by the passage. The questions do not
involve any tricks or hidden meanings.

Law School Admission Council


Law School Admission Council
Law School Admission Council
THE PREPTEST

• Analytical Reasoning .......................................SECTION I

• Logical Reasoning ...........................................SECTION II

• Reading Comprehension................................SECTION III

• Logical Reasoning ...........................................SECTION IV

Law School Admission Council


1 -8-
1
SECTION I
1
Time—35 minutes
20 Questions

Directions: Each group of questions in this section is based on a set of conditions. In answering some of the questions, it may be
useful to draw a rough diagram. Choose the response that most accurately and completely answers each question and blacken
the corresponding space on your answer sheet.

Questions 1–8 1. Which one of the following could be the order in which
the poems are read, from first through fifth?
In order to promote a newly published collection of poems, an
author will give a public reading of exactly five poems—L, (A) L, O, N, M, P
M, N, O, and P. The order in which the poems are read must (B) M, L, O, P, N
conform to the following: (C) M, O, L, P, N
M is read neither immediately before nor immediately (D) M, P, L, O, N
after N. (E) N, P, L, M, O
L is read immediately before O.
Either O or P is read second. 2. If N is read first, which one of the following could
be true?
(A) M is read fifth.
(B) O is read second.
(C) P is read fourth.
(D) L is read earlier than P.
(E) M is read earlier than P.

GO ON TO THE NEXT PAGE.

Law School Admission Council


1 3. If M is read first, which one of the following must be
1 -9-

6. If P is read fourth, which one of the following could be


1
true? true?
(A) L is read third. (A) L is read third.
(B) N is read third. (B) M is read first.
(C) N is read fifth. (C) N is read first.
(D) O is read fourth. (D) N is read fifth.
(E) P is read second. (E) O is read fifth.

4. If L is read earlier than P, which one of the following 7. Which one of the following could be true?
could be true?
(A) L is read fifth.
(A) L is read third. (B) M is read third.
(B) M is read fifth. (C) N is read fourth.
(C) N is read first. (D) O is read third.
(D) O is read fifth. (E) P is read fifth.
(E) P is read second.
8. Which one of the following CANNOT be true?
5. If O is read fifth, which one of the following must be true?
(A) M is read immediately before L.
(A) M is read first. (B) N is read immediately before L.
(B) N is read third. (C) N is read immediately before P.
(C) N is read earlier than L. (D) O is read immediately before M.
(D) N is read earlier than M. (E) O is read immediately before P.
(E) P is read earlier than N.

GO ON TO THE NEXT PAGE.

Law School Admission Council


1 -10-

Questions 9–12
1
9. Which one of the following could be the schedule of
1
performances and breaks during the concert, in order
A concert will consist of separate performances by five from the first to the seventh hour?
bands—the Loners, the Neighbors, the Rockers, the Snap, and
the Yaks. The concert will last exactly seven hours, with a 1 2 3 4 5 6 7
one-hour performance by each band and two one-hour breaks. (A) Snap break Yaks Neighbors Rockers break Loners
The concert schedule obeys the following restrictions: (B) Yaks break Neighbors Rockers break Snap Loners
The second break cannot take place immediately after the (C) Yaks break Snap Neighbors Rockers break Loners
first break. (D) Yaks break Snap Loners break Neighbors Rockers
The Yaks must perform sometime before the first break. (E) Yaks break Neighbors Snap break Rockers Loners
The Snap must perform sometime before the Rockers’
performance.
The Neighbors must perform during the hour immediately
before or else during the hour immediately after the
Rockers’ performance. GO ON TO THE NEXT PAGE.
The Loners must perform during the seventh hour.

Law School Admission Council


1 10. Which one of the following can occur during the fifth
1
12. Which one of the following must be true?
-11-
1
hour?
(A) The Neighbors perform sometime before the
(A) the first break Rockers.
(B) the second break (B) The Neighbors perform sometime before the
(C) the Yaks’ performance Snap.
(D) the Snap’s performance (C) The Rockers perform sometime before the Yaks.
(E) the Neighbors’ performance (D) The Yaks perform sometime before the
Neighbors.
11. If the Yaks perform during the second hour, which one (E) The Yaks perform sometime before the Snap.
of the following must be true?
(A) The Neighbors perform during the fifth hour.
(B) The Rockers perform during the sixth hour.
(C) The Snap perform during the fourth hour.
(D) The Snap perform during the third hour. GO ON TO THE NEXT PAGE.
(E) The first break occurs during the third hour.

Law School Admission Council


1 -12-

Questions 13–16
1
13. If Mitra, Nair, and Patel teach at the higher secondary
1
school, then which one of the following statements
A community has one secondary school and one higher must be true?
secondary school. Exactly six teachers—Khan, Lahane,
Mitra, Nair, Oberoi, and Patel—teach a subject at the two (A) Khan teaches English.
schools. Each of them teaches at either the secondary school (B) Khan teaches Science.
or the higher secondary school, but not both, and each teaches (C) Lahane teaches Science.
exactly one of the following subjects: English, Geography, or (D) Patel teaches Geography.
Science. The following is also known: (E) Patel teaches Science.
Any two of the teachers who teach at the same school do
not teach the same subject as each other.
Khan teaches at the same school as Lahane.
Lahane teaches the same subject as Mitra.
Nair teaches Science. GO ON TO THE NEXT PAGE.
Oberoi teaches English.

Law School Admission Council


1 14. If Nair teaches at the same school as Oberoi, then the
1
15. If Oberoi and Patel teach at the secondary school, then
-13-
1
teachers in which one of the following pairs must teach which one of the following statements must be true?
at the same school as each other?
(A) Khan teaches Geography.
(A) Khan and Nair (B) Khan teaches Science.
(B) Khan and Oberoi (C) Patel teaches Science.
(C) Lahane and Oberoi (D) Khan teaches at the secondary school.
(D) Mitra and Oberoi (E) Mitra teaches at the higher secondary school.
(E) Oberoi and Patel
16. If Lahane and Nair teach at the secondary school, then
which one of the following statements must be true?
(A) Khan teaches English.
(B) Khan teaches Science.
(C) Patel teaches English.
(D) Mitra teaches at the secondary school.
(E) Oberoi teaches at the secondary school.

GO ON TO THE NEXT PAGE.

Law School Admission Council


1 -14-

Questions 17–20
1
17. Which one of the following could be an accurate list of
1
the saris, from 1 through 5?
Exactly five saris are displayed in a boutique. The saris are
numbered 1 through 5, and each sari is one of three kinds: (A) Muga, Muga, Paithani, Muga, Shalu
either a Muga, a Paithani, or a Shalu. The display must be (B) Muga, Paithani, Muga, Paithani, Paithani
consistent with the following: (C) Muga, Shalu, Muga, Shalu, Shalu
At most two Paithanis are displayed. (D) Shalu, Muga, Shalu, Shalu, Muga
3 is not a Paithani. (E) Shalu, Paithani, Shalu, Shalu, Muga
2 is the same kind of sari as 5.
If 1 is a different kind of sari than 3, then 2 and 4 are both
Shalus.
If 3 and 4 are both Shalus, then 1 is a Paithani.
GO ON TO THE NEXT PAGE.

Law School Admission Council


1 18. If exactly one Shalu is displayed, which one of the
1 -15-

20. If 4 is a Paithani, which one of the following could be


1
following must be true? true?
(A) 3 is the Shalu. (A) Exactly one Shalu is displayed.
(B) 4 is the Shalu. (B) Exactly two Paithanis are displayed.
(C) 2 is a Paithani. (C) Exactly three Mugas are displayed.
(D) 1 is a Paithani. (D) Exactly four Mugas are displayed.
(E) 5 is a Muga. (E) Exactly five Mugas are displayed.

19. Which one of the following must be true?


(A) No more than one Paithani is displayed.
(B) No more than three Mugas are displayed.
(C) No more than four Shalus are displayed.
(D) At least one Muga is displayed.
(E) At least one Shalu is displayed.

S T O P
IF YOU FINISH BEFORE TIME IS CALLED, YOU MAY CHECK YOUR WORK ON THIS SECTION ONLY.
DO NOT WORK ON ANY OTHER SECTION IN THE TEST.

Law School Admission Council


2
¼ 2
¼ 2 2
-16-

SECTION II
Time—35 minutes
25 Questions

Directions: The questions in this section are based on the reasoning contained in brief statements or passages. For some
questions, more than one of the choices could conceivably answer the question. However, you are to choose the best answer; that
is, the response that most accurately and completely answers the question. You should not make assumptions that are by
commonsense standards implausible, superfluous, or incompatible with the passage. After you have chosen the best answer,
blacken the corresponding space on your answer sheet.

1. When primatologist Akira Suzuki began studying snow 2. Body temperature can be estimated by the relative
monkeys in the 1950s, he found that they often roamed proportion of two oxygen isotopes in bone. Such
out of the mountains to feed in apple orchards. After a estimates derived from dinosaur fossils suggest that
decade of observing this behavior, Suzuki began to feed dinosaurs had nearly the same body temperature in their
the monkeys in their mountain habitat by providing limbs as in the rest of their bodies. Typically, the core
them with soyabeans to eat. The monkeys no longer body temperature of cold-blooded animals today is
raided the orchards. When Suzuki began his work, much higher than the body temperature in their limbs.
23 snow monkeys lived in the region he studied. The Thus, dinosaurs were probably warm-blooded.
population today is 270 snow monkeys and is expected
Which one of the following, if true, most seriously
to continue growing.
weakens the argument?
Which one of the following claims is most strongly
(A) Large warm-blooded animals keep their core
supported by the information above?
body temperature slightly higher than the body
(A) Snow monkeys do not feed outside of their temperature in their limbs.
mountain habitat when food is readily (B) The fossilization process introduces changes to
available within it. bones such that their original oxygen isotope
(B) For snow monkeys, soyabeans provide more ratios cannot be predicted.
complete nutrition than other beans. (C) Oxygen was more abundant in Earth’s
(C) In feeding soyabeans to the monkeys, Suzuki atmosphere during the period in which the
did not intend to provoke the phenomenal dinosaurs lived.
population growth that resulted. (D) Small warm-blooded animals like mice tend to
(D) Snow monkeys eat apples only if there is no have more uniform body temperatures than do
other fruit to eat. large warm-blooded animals like elephants.
(E) Feeding soyabeans to snow monkeys has proved (E) Warm-blooded animals are more active and use
to be an environmentally unsound policy. more oxygen than cold-blooded animals.

GO ON TO THE NEXT PAGE.

Law School Admission Council


2
¼ 2
¼ 2 2
-17-

3. Psychological depression is a physiological disorder Questions 5–6


over which the person suffering has no conscious
Mehra: Rao’s analytic concepts are wrong and should
control. Therefore, no one who suffers from
be rejected. As a psychoanalyst myself, however, I can
psychological depression can properly be considered
understand why certain psychoanalysts adhere to them.
morally weak because of a supposed inability to feel
These psychoanalysts acquired their “emotional
better through force of will.
certainty” that Rao’s views are correct while training
The conclusion follows logically if which one of the under her. This training includes one’s own
following is assumed? psychoanalysis, in which the teacher interprets the
actions, dreams, and fantasies of the student in analytic
(A) Moral weakness is commonly thought to be
terms. Strong emotional bonds with the teacher are
caused by psychological depression.
formed, bonds that predispose the student to accept the
(B) People who cannot control their feelings
teacher’s analytic concepts as correct. It is thus
consciously cannot properly be considered
impossible for a student to make unbiased judgments
morally weak.
about the value of the teacher’s analytic concepts.
(C) An ability to pull oneself out of psychological
depression is not necessarily a sign of moral
5. Based on the information in the passage, which one of
strength.
the following is the most accurate assessment of Mehra’s
(D) Some people who suffer from psychological
claim that Rao’s analytic concepts are wrong and should
depression are nevertheless able to feel better
be rejected?
through force of will.
(E) People who feel morally weak always feel a (A) The claim is dubious because Mehra assumes
loss of conscious control. without justification that she is qualified to
judge Rao’s analytic concepts.
4. Business writer: Although the demand for Corinne (B) The claim has not been established because
wristwatches currently far outstrips supply, it Mehra provides evidence that could show only
would be a mistake for the manufacturer to that Rao’s students are biased in the evaluation
increase supply to meet demand. The current of Rao’s analytic concepts.
demand results from the public’s perception that (C) The claim cannot be evaluated because it is not
the wristwatches are in short supply, and the possible for any psychoanalyst to provide an
wristwatches are in short supply merely because objective assessment of another
they are produced in very limited quantities. psychoanalyst’s analytic concepts.
The excess demand creates the impression that (D) The claim is questionable because it is obvious
the wristwatches are greatly desired, and that that Mehra has a professional rivalry with Rao
impression in turn helps account for consumers’ and cannot judge Rao’s concepts fairly.
desire for them. If the supply of Corinne (E) The claim is acceptable because Mehra has
wristwatches were to increase to meet demand, effectively shown that Rao’s analytic concepts
excess demand for them would be eliminated, are biased and based on emotion rather than
with the result that the wristwatches would no rational thinking.
longer be desired.
6. Suppose it were learned that several of Rao’s students,
The claim that it would be a mistake for the manufacturer
upon completion of their training, rejected her analytic
of Corinne wristwatches to increase supply to meet
concepts. This would
demand plays which one of the following roles in the
business writer’s argument? (A) demonstrate that Mehra is correct in rejecting
Rao’s analytic concepts
(A) an introductory claim that describes the
(B) cast doubt on Mehra’s description of what
position to be refuted by the argument
happens as a result of psychoanalytic training
(B) a justification of the relevance of the
under Rao
evidence cited
(C) demonstrate that Rao is not an effective teacher
(C) a premise of the argument
of psychoanalysis
(D) an opinion offered in support of one of the
(D) demonstrate Mehra’s claim that Rao should
argument’s premises
change her method of teaching
(E) the conclusion toward which the argument
(E) cast doubt on the belief that psychoanalytic
is directed
training can be worthwhile for a student

GO ON TO THE NEXT PAGE.

Law School Admission Council


2
¼ 2
¼ 2 2
-18-

Questions 7–8 9. Editor: Evidence shows that restrictions on tobacco


advertising have had a significant impact on
Mr. Hussain: The arctic squirrel gets so cold while
smoking among adults. A recent survey has
hibernating that its blood temperature falls well below
shown that a smaller percentage of adults now
the temperature at which water freezes. Although the
smoke than at any other time in the last two
squirrel’s blood is about 70 percent water, the blood
decades. The decline in the percentage of adults
never freezes while the squirrel is hibernating. The
who smoke has been most marked during the last
squirrel’s blood, therefore, must contain a substance
ten years, and, not coincidently, some of the most
that prevents the blood from freezing at the temperature
important restrictions on tobacco advertising
at which water freezes.
came into force ten years ago.
Ms. Shah: The blood contains no such substance. Laboratory
experiments involving a number of animals, including The reasoning in the editor’s argument is most
the arctic squirrel, have shown that a vial of blood from vulnerable to criticism on the grounds that the argument
any of the animals freezes at just the same temperature (A) fails to consider whether there have been any
as does a vial of water. changes over the last two decades in the
percentage of the teenage population who smoke
7. Ms. Shah’s response proceeds by (B) uses evidence that describes only a percentage
(A) presenting evidence that supports a conclusion of the adult population to reach a conclusion
inconsistent with Mr. Hussain’s conclusion about the entire adult population
(B) showing that the evidence offered by (C) reaches a conclusion about smoking among
Mr. Hussain was collected by means of today’s adults based on statistics from ten or
unreliable methods twenty years ago
(C) offering an alternative explanation for why the (D) neglects to take into account whether there have
squirrel’s blood fails to freeze at the been restrictions on the advertising of other
temperature at which water freezes products besides tobacco in the past ten years
(D) showing that a key term used by Mr. Hussain (E) fails to consider the possibility that factors other
is ambiguous than restrictions on advertising have contributed
(E) showing that the evidence provided by to the decline in smoking among adults
Mr. Hussain has no bearing on the point
at issue

8. Ms. Shah’s response depends on the assumption that


any substance normally in the squirrel’s blood that GO ON TO THE NEXT PAGE.
might keep the blood from freezing at the freezing
point of water
(A) cannot be detected without extracting blood
from a living squirrel
(B) would have remained in the blood while the
experiment was being performed
(C) would be a component in the blood of any
animal that survives arctic winters
(D) would also be present in the squirrel’s other
bodily fluids
(E) has already been identified in published research

Law School Admission Council


2
¼ 2
¼ 2 2
-19-

10. Medical researcher: Studies in North America have 12. Politician: Tightening air quality standards that regulate
shown that the incidence of heart disease in a industrial emissions would cause industries to
population is closely related to the average fat move to locations with less stringent standards
consumption for individuals in that population. concerning these emissions. So current standards,
However, although residents of France consume, which are already quite stringent, should not be
on average, as much fat as residents of raised, since there is not enough evidence that
North America, heart disease presently occurs the decreased pollution that would admittedly
half as frequently among the French as among result from raising the standards would
North Americans. compensate for the loss of jobs caused by the
relocation of key industries.
Which one of the following, if true, would most help
to resolve the apparent conflict noted by the medical Which one of the following principles, if valid, most
researcher? helps to justify the politician’s reasoning?
(A) The average level of fat consumption by the (A) Governmental policy should generally be
French has been falling for several decades. designed to encourage the growth of key
(B) Other factors of diet besides high consumption industries.
of fat have not been similarly linked with (B) The extent to which people are willing to
incidence of heart disease. accept a governmental policy should be the
(C) Heart disease takes years to develop and the only factor determining whether that policy
average level of fat consumption in France is adopted.
increased to North American levels only a (C) Governmental policy should be altered only
few years ago. if there is compelling evidence that the
(D) Certain diseases other than heart disease have consequences of doing so are better than the
also been linked to average fat consumption, consequences of not doing so.
and the French have a higher incidence of (D) Governmental policy should be changed in
these than do North Americans. those cases in which retaining the policy and
(E) Cigarette smoking significantly increases the changing it each have positive and negative
risk of heart disease and France has a higher consequences.
percentage of cigarette smokers in its (E) If one lacks clear evidence about whether a
population than does North America. given action will have a specified consequence,
then one should assume that it will have that
11. Professor Suri: Professor Khanna’s report characterizes consequence when deciding whether to
our colleague Professor Yadav as too flamboyant perform that action.
and confrontational in the classroom. But the
argument given in this report sounds so 13. Santayana recommends that we study history to avoid
much like the classic argument of the self- the mistakes of the past. But we should not follow his
serving academic that one has to wonder if advice. For, since history consists of unique and
Professor Khanna simply is not the orator and unrepeatable accidents, none of the crises we now
entertainer that Khanna would like to be, and so face are the same as those our ancestors faced. Thus,
vents frustrations by condemning others— studying history never enables one to avoid mistakes
Professor Yadav in particular. of the past.
Professor Suri’s argument is flawed because it Which one of the following most accurately expresses
the main conclusion of the argument above?
(A) confuses the distinction between being overly
confrontational and engaging students by (A) People should not study history, since doing so
entertaining them leads them to misunderstand the crises they
(B) presupposes the point it is attempting to establish now face.
(C) mistakes Professor Khanna’s characterization of (B) Every historical period is different from every
a view for an endorsement of that view other historical period.
(D) attacks Professor Khanna personally instead of (C) Although the crises one generation faces may
addressing Professor Khanna’s argument appear to be the same as those another
(E) rejects the possibility that Professor Yadav is in generation faces, they never are.
fact too confrontational (D) Studying history is valuable, but not for the
reason that Santayana suggests.
(E) One should not try to avoid repeating the
mistakes of previous generations by
studying history.

GO ON TO THE NEXT PAGE.

Law School Admission Council


2
¼ 2
¼ 2 2
-20-

14. When polar ice caps grow (during ice ages, for 16. A recent study found that small rats were
example), lighter forms of oxygen from water vapor approximately twice as likely, and large rats only
and seawater accumulate in the frozen ice caps, leaving half as likely, to suffer from heart problems than were
greater concentrations of one heavy form of oxygen rats of average size.
behind in the sea, where it is absorbed by marine
Each of the following, if true, contributes to an
organisms. When and as the ice caps shrink, the
explanation of the correlation given above between
concentrations of this heavy oxygen in seawater
size and heart disease in rats EXCEPT:
decrease. During one 30,000-year period, concentrations
of this heavy oxygen in sea shellfish increased for about (A) Small rats are more likely than large rats to
20,000 years, then decreased for 10,000 years. suffer from fatal diseases that tend to strike
earlier than do heart problems.
The information given most strongly supports which
(B) Small rats generally have smaller blood vessels
one of the following hypotheses about the period
than do large rats, vessels that can more easily
described above?
be clogged with fatty deposits.
(A) Average global temperatures 10,000 years (C) The larger a rat is, the more successful it will
after the beginning of the period be at defending itself, and therefore the less
approximately equaled average global stressful its existence will be.
temperatures 20,000 years later. (D) In addition to being the leading cause of weak
(B) Polar ice caps at the beginning of the period hearts in adult rats, malnutrition at early stages
were larger than they were at the end of of development causes rats to be undersized.
the period. (E) Although large rats are no more active than rats
(C) The beginning of the period coincided with the of average size, they tend to be much more
onset of an ice age that lasted approximately active than small rats, resulting in greater
20,000 years. cardiovascular conditioning.
(D) The polar ice caps grew for about 20,000 years
after the period began, then began to shrink. 17. To measure a small boat hull (the main body that rides
(E) An ice age was drawing to an end during the in the water) accurately, it is necessary to level the hull
first 20,000 years of the period. so that a baseline can be established. This does not
require sophisticated tools, but it does require a leveling
15. People would not follow a leader if they felt that there tool called a “line level.” When scientists discovered a
was nothing they could gain by following that leader. small ninth-century boat, they wanted to record the
Therefore, even those leaders who are incompetent or dimensions of its hull accurately. None of the scientists,
evil bring some good to their followers. however, had any experience measuring hulls. Because
of this they were not sure they had leveled the hull
The flawed reasoning in the argument above is most
before establishing a baseline.
similar to that in which one of the following?
If the statements above are true, then which one of the
(A) Because people expound upon only theories
following must also be true?
they believe are true, any theory that is
expounded upon by someone contains at least (A) The scientists did not accurately measure the
a grain of truth. hull’s dimensions because they had no
(B) Because there is some good to be found in experience measuring hulls.
even the worst circumstances, the world’s (B) The scientists accurately determined the hull’s
most vicious people must have brought about dimensions, provided that they leveled the hull.
at least some good. (C) If the scientists did not accurately determine the
(C) To be a worthy leader, one must bring some hull’s dimensions, it was because they did not
good to people. Therefore, those who bring have a line level.
about some good make worthy leaders. (D) The scientists were able to accurately record
(D) Because people can never completely separate the hull’s dimensions only if they used a
what is true from what they wish to be true, line level.
no theory is ever completely true. (E) If the scientists had measured the hull’s
(E) Even leaders of scientific revolutions are dimensions accurately, then at least one of them
influenced by the false theories of their would have had experience measuring hulls.
predecessors. Therefore, the influence of earlier
false theories never completely disappears.

GO ON TO THE NEXT PAGE.

Law School Admission Council


2
¼ 2
¼ 2 2
-21-

18. Desai: The municipal council recently passed a 20. Economist: When national governments dispense funds
rent-control ordinance. However, a recent study to local governments to spend on local projects,
of fifteen local municipalities shows clearly that any local government can receive a greater
rent control increases the price and lowers both proportion of government funds by creating more
the quality and the availability of rental units. local projects than other local governments create
Thus, it can be concluded that it is not the on average. Due to this added incentive to create
municipal council’s objective to preserve the more local projects, overall government spending
quality and availability of local rentals. and taxation are greater everywhere than they
would be if local governments funded projects
Desai’s conclusion follows logically if which one of the
entirely by themselves.
following is assumed?
Which one of the following most closely conforms to
(A) The recent study of local rent-control
the proposition illustrated by the economist’s statements?
ordinances was conducted by impartial
investigators. (A) A large company invests in a new technology
(B) Rent control is not an appropriate topic for that greatly improves its product. Smaller
consideration by the municipal council. companies, individually unable to match the
(C) The councillors who voted for rent control large company’s investment, pool their
agree with the study’s conclusions about resources and invest in the new technology
rent control. in order to compete.
(D) Some councillors who voted for rent control (B) A national government finances an irrigation
stand to profit from rent control. project in order to turn an arid valley into
(E) The municipal council sometimes acts in an fertile farmland. The food grown in the valley
arbitrary and irrational manner. reduces local prices, which helps consumers
in the valley but hurts farmers elsewhere.
19. Geologist: As a spinning object’s radius decreases, its (C) A sales manager offers prizes for the salesperson
rate of rotation increases; for example, figure who sells the most products during a month.
skaters can twirl faster by bringing their arms This causes each salesperson to try to undercut
closer to their sides. This law of physics helps the prices of the other salespeople, and as a
to explain why, over the past decade, Earth’s rate result profits are lower than they would
of rotation increased. For earthquakes have otherwise be.
increased in number and severity during the past (D) People pool their money to buy certain foods
decade; and since immediately after an in bulk in order to get a volume discount and
earthquake Earth’s tectonic plates move toward reduce their bills. But after doing this, they eat
the center of the earth, their movement causes more than they used to and so spend the same
Earth’s radius to decrease. amount of money on food as they did before
Which one of the following, if true, would most they pooled it.
undermine the geologist’s explanation of why Earth’s (E) Several communities band together to build a
rate of rotation increased over the last decade? large conference center. But the project nearly
collapses as each community, because of the
(A) In the beginning of the twentieth century, revenue the center will bring, fights to be the
geologists observed an increase in number one in which it is built.
and severity of earthquakes, accompanied
by a decrease of Earth’s radius.
(B) During the past decade, other geological events
have counteracted the movements of the tectonic
plates that occur immediately after earthquakes.
(C) Only skillful figure skaters succeed in GO ON TO THE NEXT PAGE.
twirling faster by bringing their arms closer
to their sides.
(D) Since the time of the ancient Egyptians, Earth’s
rate of rotation has been known to fluctuate.
(E) Increased volcanic activity over the last ten
years suggests that the overall movement of
the tectonic plates has increased.

Law School Admission Council


2
¼ 2
¼ 2 2
-22-

21. Philosopher: For some kinds of art, there is truth to the 23. When a resource’s price reflects its full cost to society,
adage that beauty is in the eye of the beholder. consumers pay directly for externalities—the unintended
A painting or sculpture that strikes one person but harmful consequences to society of using a resource.
as having artistic value may strike another as Further, the price of a resource deters misuse of that
lacking such value. Consequently, there are no resource only if it reflects the resource’s externalities.
valid objective standards for determining the So, since sound management of a resource requires that
artistic value of a painting or a sculpture. the price of the resource deter its misuse, it also requires
that the price reflect whatever unintended harm to the
The philosopher’s reasoning is flawed because it
environment the use of that resource causes.
(A) relies exclusively on an old adage whose truth
The conclusion follows logically if which one of the
has not been demonstrated
following is assumed?
(B) ignores the fact that there are other kinds of art,
such as literature and poetry, that may also (A) Whatever constitutes unintended harm to the
occasion disagreement environment also constitutes unintended harm
(C) fails to consider whether there are valid to society.
objective standards for evaluating nonvisual (B) A resource’s externalities are always taken into
art, such as music account in setting the price of that resource.
(D) fails to consider that people who disagree about (C) When setting a resource’s price, it is possible
the artistic value of a given painting may be to forecast completely and accurately all of
incorrectly applying the same evaluation the effects of using that resource.
criteria to that painting (D) If a resource is soundly managed, then its
(E) fails to consider that the individuals judging externalities are precisely assessed.
and disagreeing about works of art may be (E) The price of a given resource is only one of
experts in making such judgments several factors relevant to a decision about
whether to use it.
22. Researcher: Experiences that are accompanied by
increased secretions of adrenaline—a hormone
produced in situations involving fear—tend to be
remembered more clearly than experiences not so
accompanied. Thus, the details of frightening
experiences tend to be remembered more clearly
GO ON TO THE NEXT PAGE.
than do the details of nonfrightening experiences.
Which one of the following, if true, most seriously
weakens the researcher’s argument?
(A) Some experiences are so intense that an
individual’s normal tendency to retain the
details of them is reversed.
(B) An individual will tend to remember most
clearly those details of a situation that are
relevant to the satisfaction of desires.
(C) Highly pleasurable experiences, like frightening
experiences, are accompanied by increased
levels of adrenaline.
(D) Frightening experiences make up only a small
fraction of experiences in general.
(E) If an individual perceives a dangerous situation
as nonfrightening, then the experience of that
situation will not be accompanied by increased
adrenaline secretions.

Law School Admission Council


2
¼ 2
¼ 2 2
-23-

24. Researcher: Several chemical compounds found in 25. According to the Newtonian theory of gravity, the
soyabeans are effective agents for cancer gravitational force between two bodies depends
prevention in humans. These include isoflavones exclusively on their mass and the distance between
and phytosterols, which also occur in other plants them, but is independent of what the bodies are made
but are especially concentrated in soyabeans. In of. Thus, according to the Newtonian theory, one does
rats and larger mammals such as cheetahs and not need a theory of the structure and constitution of the
sheep, isoflavones have been shown to inhibit the Sun and the planets in order to calculate their orbits.
production of estrogen, and estrogen is known to
The conclusion of the argument above follows logically
promote certain types of cancer. Phytosterols may
if which one of the following is assumed?
slow cholesterol absorption and thus have
anticancer effects. (A) Mass is dependent on gravitational force.
(B) According to the Newtonian theory, the
Each of the following, if true, would reduce the support
calculation of planetary orbits requires
given by the researcher for the claim that several
considering only their gravitational forces.
chemical compounds in soyabeans are effective agents
(C) The Newtonian theory of gravity is mistaken
for human cancer prevention EXCEPT:
about the data needed to calculate gravitational
(A) The soyabean derivatives used in most studies force.
are the products of specialized processing (D) Knowing what an object is made of is sufficient
techniques and are not yet widely available for determining its mass.
to consumers. (E) The gravitational force between the Sun and a
(B) While phytosterols, which occur in high planet is a factor in determining the distance
concentrations in soyabeans, have been shown between them.
to decrease cholesterol absorption in the body,
new evidence suggests that this decrease is not
large enough to reduce susceptibility to cancer.
(C) A study of people with high levels of blood
cholesterol showed no significant reduction
in cholesterol levels after switching to a
soyabean-rich diet.
(D) Consumption of soyabean products might lower
blood cholesterol in animals, but a study of
people whose major source of protein is
soyabeans rather than animal products showed
blood cholesterol levels no lower than normal.
(E) Preliminary studies have not shown isoflavones
to have a significant effect on estrogen levels
in humans.

S T O P
IF YOU FINISH BEFORE TIME IS CALLED, YOU MAY CHECK YOUR WORK ON THIS SECTION ONLY.
DO NOT WORK ON ANY OTHER SECTION IN THE TEST.

Law School Admission Council


3 -24-
3 3
SECTION III
3 3
Time—35 minutes
24 Questions

Directions: Each set of questions in this section is based on a single passage or a pair of passages. The questions are to be
answered on the basis of what is stated or implied in the passage or pair of passages. For some of the questions, more than one
of the choices could conceivably answer the question. However, you are to choose the best answer; that is, the response that
most accurately and completely answers the question, and blacken the corresponding space on your answer sheet.

English author Virginia Woolf, who published her psychological theme, but rather in Woolf’s effort to
major works in the 1920s and 1930s, is generally mold a number of such themes into structures that
considered a significant figure in literary modernism— might make lasting sense of the fluidity of life. By
a movement marked by a pronounced break with (55) orchestrating the workings of consciousness and the
(5) traditional forms and techniques of expression. Like quality of personal relationships, Woolf endeavors to
other modernists Woolf abandoned established notions distill the fundamental experience of living.
of character development and plot; indeed, her major
novels contain no substantial narrative impulse. Not 1. Which one of the following most accurately expresses
surprisingly, her experiments in novel writing were not the main point of the passage?
(10) readily accepted by traditional critics, who complained
(A) While some literary critics have faulted Woolf’s
that her novels lacked vital, well-defined characters,
work for lacking well-defined characters,
narrative development, and dramatic force.
dramatic plots, or political themes, these
But what these critics saw as shortcomings were
choices result from Woolf’s modernist
actually part of a considered aesthetic rationale
(15) derived from Woolf’s adaptation of the conventions approach to human character and novelistic
of modernism to express her own interests in the form.
psychological dimensions of existence. Woolf had (B) Even though Woolf is a significant figure in
ceased to believe in clearly definable human character; literary modernism, recent critics claim that her
she felt that people’s knowledge of one another was work lacks impact because it does not address
(20) necessarily incomplete. As a result, Woolf rejected the issues that have social and political
device of an omniscient narrator—a narrator who has ramifications.
unimpeded knowledge of the interior life of every (C) While Woolf abandoned traditional ideas of
character in the novel. Instead of resorting to the character and plot in writing her novels,
illusion of the all-seeing eye, Woolf presents her traditional critics concede that her novels
(25) readers with brief glimpses of the impressions that succeed because they distill the fundamental
characters receive from each other. The characters’ experience of living.
perceptions are so thoroughly interwoven in the (D) Woolf believed that people’s knowledge of one
pattern of the novel that it becomes difficult to gain a another was incomplete and that it was thus
detailed picture of any one character from isolated impossible for characters portrayed in novels
(30) passages; portraits of characters emerge only from the to have clear insight about one another.
cumulative effect of the whole. Analogously, for (E) Literary critics have typically viewed Woolf’s
Woolf, tragedy consisted not in violent upheavals but major works as experimental in nature because
in unexpected and painful recognitions, the modest of her modernist approach, but these critics are
epiphanies embedded in daily life; so Woolf chose to divided among themselves about the value of
(35) reverse the approach of conventional fiction—she her works.
emphasized the rhythms of the normal moments of life
and de-emphasized the more dramatic ones. 2. Which one of the following does the passage identify as
Because Woolf’s method of characterization a method Woolf used to portray characters in her
entails depicting characters through the shifting novels?
(40) perceptions of other characters, she concentrated on
those kinds of people from whose eyes she could (A) construction of intricate plots
imagine herself looking out upon the world—namely, (B) inclusion of extensive dialogues
characters with her own middle-class social and (C) development of highly dramatic scenes
economic status. Some recent critics charge that (D) presentation of glimpses of characters’
(45) because she based her technique on the psychology perceptions of one another
and perceptions of her characters, Woolf did not (E) use of the device of the omniscient narrator
address issues that are more broadly public, social,
and political in nature. But her strict fidelity to the
complexity of human perceptions prevents her novels
(50) from degenerating into the narrowly private. The
center of a Woolf novel does not reside singly in some GO ON TO THE NEXT PAGE.

Law School Admission Council


3 3
3. The passage provides information that is most relevant
3 3
5. In the passage, the author primarily seeks to
-25-
3
to answering which one of the following questions?
(A) illuminate one author’s artistic approach by
(A) What elements of the modernist aesthetic did defending it against various criticisms leveled
Woolf reject? against the author’s work
(B) For what reasons did modernist writers abandon (B) identify the major precepts of a literary
established notions of character and plot? movement by examining the work of a
(C) What was Woolf’s view of literature that prominent figure within that movement
attempted to be more explicitly political than (C) examine how one author’s work demonstrates
her own work? that traditional techniques of fiction writing
(D) How did negative criticism of Woolf’s work have become obsolete
affect the development of her aesthetic approach? (D) show that the criticisms directed at one author’s
(E) Why does Woolf’s work lack explicit portraits work are typical of the kinds of criticisms
of her characters? directed at experimental writers
(E) discuss the relation of the works of one writer
4. According to the passage, Woolf chose not to use an to the larger literary movement to which
omniscient narrator to guide readers’ perceptions of her she belonged
characters because of her belief that people
6. By “the illusion of the all-seeing eye” (lines 23–24), the
(A) act in ways that are often inconsistent and
author of the passage primarily means the use in a novel
impossible to understand
of a narrator who
(B) have only limited knowledge of one another
(C) experience their most meaningful revelations in (A) provides a thoroughly impartial account of the
normal, daily existence events that make up the novel
(D) typically make straightforward judgments about (B) makes explicit statements regarding the
one another narrator’s own motivations and feelings
(E) share certain fundamental psychological traits (C) relies heavily on visual imagery in describing
characters and events
(D) can provide insight into the thoughts and
feelings of any character in the novel
(E) can predict the outcomes of the novel’s plot
developments

7. According to the passage, Woolf held that tragedy


resides in
(A) the indefinability of human character
(B) the small revelations that are a part of daily living
(C) the incomplete knowledge that people have of
one another
(D) the complexity of human perceptions
(E) the circumscribed experiences of members of
her social class

GO ON TO THE NEXT PAGE.

Law School Admission Council


3 -26-
3
Much has been written about the importance of
3 3
8. Which one of the following most accurately expresses
3
the church as a social and political force in the lives the main idea of the passage?
of many African Americans. However, historians are
(A) Although the extent of the role played by African-
only beginning to uncover the important role played
(5) by African-American women in establishing the American women in the Baptist church has
church’s significance, particularly that of the Baptist long been a subject of debate, recent studies
church in the early years of the twentieth century. confirm the significance of their contributions
During this period, many African-American both to the church and to society at large.
Baptists turned more than ever to the church as a (B) The programs initiated by African-American
(10) means of nurturing a sense of community and of Baptist women succeeded primarily because
organizing resistance to the discriminatory practices they coincided with a period in which members
of the majority culture. In order to achieve the former were beginning to look to the church to meet
end, they sought to situate the church as a sphere in needs that the larger society had failed to satisfy.
which values and issues could be aired, debated, and (C) While the role of the Baptist church in African-
(15) ultimately disseminated throughout the community as American life has long been recognized,
a whole. To achieve the latter end, members operated scholars have only recently begun to recognize
vital services such as employment bureaus, savings the influence that African-American Baptist
banks, day care centers, and health clinics under the women have had on the church.
church’s aegis. Scores of African-American women (D) The social activism of African-American
(20) were responsible for the creation and maintenance of women early in the twentieth century was a
these services. In addition, an autonomous national result of efforts by the Baptist church, which
women’s convention was organized, which in turn helped raise their consciousness by involving
founded a national training school for the education of them in public discussion of social and
young African-American Baptist women. To all these political issues.
(25) endeavors the women brought a rising consciousness (E) While the role of African-American Baptist
of their position and potential in the larger society, a women in providing social services for church
consciousness in many cases forged or elaborated members has long been acknowledged, it is
upon in the context of the public discussions of social only recently that historians have discovered
issues at individual churches. the vital part that these women played in
(30) Some historians postulate that these years were shaping church theology.
fruitful for activism not despite, but because of, the
circumstances African Americans were experiencing; 9. Which one of the following is the primary function of
in other words, the difficulties of the times created an the last paragraph?
environment that both empowered many African-
(35) American women to speak out and motivated them to (A) to provide an additional example of the
pursue social change. The Baptist women’s movement influence of African-American women on
was also notable for its ability to transcend racial and the Baptist church
gender lines. This ability is illustrated by the activities (B) to explain how African-American Baptist
of women’s missionary groups composed of seminary women were able to change the direction of
(40) graduates who traveled to African-American the Baptist church
communities giving religious and moral instruction (C) to dispute those historians who claim activism
and who frequently developed productive alliances flourished because of the challenges facing
with other missionary groups made up of African Americans
African-American men or white women and men. (D) to explain why the historians mentioned in the
(45) Perhaps historians’ most fascinating discovery first paragraph were slow to recognize how
is of the role played by African-American Baptist African-American women influenced the
women in shaping church theology. The writings of Baptist church
church leaders such as Virginia Broughton reveal a (E) to demonstrate how the expansion of women’s
concerted attempt to articulate interpretations of scripture roles resulted in greater democracy in the
(50) that would expand women’s roles and possibilities Baptist church
within and outside the church. One interesting aspect
of these writings is that, despite their gender
consciousness, they do not detract from but rather
emphasize the need to foster an atmosphere in which
(55) a truly democratic religious community can thrive.
As much can be said of the other efforts of African- GO ON TO THE NEXT PAGE.
American women to strengthen the Baptist church.

Law School Admission Council


3 3
10. According to the author, women’s missionary
3 3
14. All of the following are mentioned in the passage
-27-
3
groups serve as evidence of how African-American as contributions made by African-American Baptist
Baptist women women EXCEPT:
(A) combatted discriminatory social institutions (A) educating young women
(B) brought political change to local communities (B) lobbying for civil rights
(C) expanded economic opportunities for young (C) providing financial services
people (D) teaching religious values
(D) overcame the boundaries between different (E) interpreting scripture
races and genders
(E) worked together with representatives of other 15. The author ascribes “gender consciousness” (lines 52–53)
faiths to the writings discussed in the last paragraph most
likely because these writings
11. The author describes the views of historians in
(A) tried to interpret scripture in a way that would
lines 30–36 primarily in order to
advance the cause of women
(A) identify the social changes achieved by (B) attempted to replace scripture with the writings
African-American Baptist women of female theologians such as Virginia Broughton
(B) present an alternate interpretation of the causes (C) argued that women had played a significant role
for activism by African-American Baptist women in shaping early church doctrine
(C) illustrate the difficulties facing African (D) encouraged missionary activity as a means of
Americans at the start of the twentieth century fostering collaboration between men and women
(D) explain why African Americans were drawn to (E) criticized men for using church policy to limit
the Baptist church instead of to other churches the possibilities available to women
(E) point out an example of African-American
women’s increasing awareness of their potential 16. It can be inferred from the passage that the public
discussions held at a number of African-American
12. It can be inferred from the passage that the Baptist churches most likely resulted in which one of
African-American Baptist church in the early twentieth the following?
century provided some of its members with each of the
(A) the multiplication of factions each of which
following EXCEPT:
was authorized to pursue its own version of
(A) assistance in gaining political office social activism
(B) opportunities for higher education (B) an improved sense of community with members
(C) a forum for discussing social issues of the majority culture
(D) a religious justification for social change (C) a greater reliance on Baptist church leaders to
(E) support in the struggle for women’s rights devise and implement solutions to social-
service problems
13. The primary purpose of the passage is to (D) an increased ability on the part of the Baptist
(A) discover the causes of a particular phenomenon church to work with other Baptist organizations
(B) measure the effects of a particular phenomenon (E) an enhanced awareness by some Baptist church
(C) summarize new findings about a particular members of their ability to affect conditions
phenomenon in society
(D) defend the prevailing view of a particular
phenomenon
(E) compare factors that influenced a particular
phenomenon
GO ON TO THE NEXT PAGE.

Law School Admission Council


3 -28-
3
Both of the following passages comment on a
3
(55)
3
was to be conveyed to England. By rejecting the
3
particular nineteenth-century English court case. seemingly straightforward conclusion of breach of
Passage A contract, the Raffles judges transformed a single
When faced with issues of uncertainty in lawyer’s momentary and apparently desperate
contracts, the courts attempt to ascertain the intention manipulation into a precedent-setting rule of law.
of the parties as disclosed in the contract document.
17. It can be inferred that both authors would be likely to
Ordinarily, this determination depends on an objective
agree with which one of the following statements about
(5) test: what would a reasonable observer have thought
Raffles v. Wichelhaus?
that this meant? When the objective test yields no
result, it is usually because the wording of the contract (A) It is a case in which all objective indications
is simply unintelligible. But in the 1864 landmark case suggest that the contracting parties had never
Raffles v. Wichelhaus, the objective test is inapplicable achieved a meeting of the minds regarding the
(10) for a slightly different reason: it was not possible to issue in dispute.
determine what a crucial expression in the contract (B) It is of interest to legal scholars and historians
referred to. At issue was a contract for the sale of some chiefly because of what it reveals about the
cotton described as “ex Peerless from Bombay,” legal underpinnings of international trade in
meaning that it was to be transported from India to the nineteenth century.
(15) England in a ship named Peerless. However, there (C) It had important implications for subsequent
were two ships named Peerless sailing from Bombay, legal cases involving issues of uncertainty
one in October and the other in December. The in contracts.
defendant claimed he understood that he was buying (D) The dispute at its heart arose because the
cotton to arrive on the October ship. The seller document at issue used language that was
(20) understood the cotton in question to be that carried in
fundamentally incoherent.
the December ship. When the latter arrived, the buyer
(E) The particular kind of ambiguity around which
refused to accept it.
its central issues revolved is typical of most
The wording of the contract provides no basis
court cases involving contract disputes.
for a reasonable observer to reach a conclusion
(25) about which cotton was being bought and sold. In a
18. It can be inferred that both authors would be likely to
precedent-setting decision, the court ruled in the
agree with which one of the following statements
defendant’s favor, accepting his argument that there
regarding Raffles v. Wichelhaus?
had been no meeting of the minds in the bargaining
process. Normally this kind of argument cannot be (A) The seller was aware that there were two ships
(30) used, because it depends on a subjective test. But in named Peerless, but nonetheless failed to
cases of this sort, where the objective test yields no specify one over the other.
decision, there really is no alternative but to use a (B) At the time the contract was signed, the
subjective test. defendant was aware that there would be two
Passage B cotton shipments matching the description
At the time of Raffles, prevailing contract law “ex Peerless from Bombay.”
(35) treated contractual obligation as arising from internal (C) Even if a different set of judges had tried the
psychological conditions of agreement whose case, the ruling probably would have been
verification depended on external signs. In this case, the same.
though, these signs were only contingently and (D) The precedents set by the case have been
ambiguously linked to the internal mental states. invalidated by subsequent judicial rulings.
(40) What is striking about this decision is how easily it (E) The disputed contract did not state precisely
could have been decided differently: a cursory reading when the cotton in question was to be shipped
of the case offers ample evidence that a steep decline from Bombay.
in the price of cotton had simply made the price the
defendant had negotiated earlier economically
(45) disadvantageous to him.
In admitting as plausible the defendant’s
suspiciously baroque account of how he had come to
GO ON TO THE NEXT PAGE.
believe he was not obligated to accept the cotton
delivered to him on Peerless, the judges displayed a
(50) willfulness of their own, derived from, though
certainly not reducible to, the pointed willfulness of
the defendant’s claim that what motivated his refusal
to pay was not the precipitous drop in cotton prices,
but rather, a confusion over how and when the cotton

Law School Admission Council


3 3
19. Passage B, unlike passage A, seeks to advance its
3 3
20. Each passage suggests that which one of the following
-29-
3
argument by statements is true concerning Raffles v. Wichelhaus?
(A) drawing conjectures about unacknowledged (A) The written contract for the sale of the cotton
motivations on the part of the defendant in question stated that the delivery date would
(B) challenging the presupposition that it was be negotiated later.
possible to apply an objective test in the case (B) The buyer contacted the seller sometime
in question between October and December of the year
(C) questioning the legitimacy of the grounds for in which the cotton was to be shipped.
the plaintiff’s claims (C) The seller did not ship any cotton from India
(D) using the case under discussion as an to England on the ship called Peerless that
illustration of broad principles of contract law sailed in October.
(E) expressing skepticism regarding the possibility (D) The buyer did not pay in full for the cotton in
of ascertaining the assumptions of parties to question at the time at which the two parties
contract disputes entered into their agreement for its sale.
(E) The seller resided in India at the time at which
at least one of the ships called Peerless sailed
from Bombay.

GO ON TO THE NEXT PAGE.

Law School Admission Council


3 -30-
3
The discovery in the 1930s of superconductivity—
3
(60)
3
for making most of the benefits of SQUIDs—and,
3
the ability of certain chemical elements (such as lead potentially, the other practical applications of
or tin), alloys, and compounds to offer absolutely no superconductors—widely accessible to the
resistance to the flow of electricity through them when general public.
(5) they are cooled to very low temperatures—raised hope
among physicists about the prospect of putting 21. Which one of the following most accurately states the
superconducting substances to countless practical uses main point of the passage?
that would increase energy efficiency in everyday
(A) Unless there are significant breakthroughs in
electrical machines such as computers and trains.
their manufacture, superconductors are
(10) But, despite a few successful applications, such
unlikely to revolutionize the computer and
developments have generally not arrived. The main
transportation industries.
reason for this is that it is both difficult and costly to
cool these materials to the extent necessary for them to (B) Despite drawbacks such as brittleness,
reach and maintain their transition temperature—the high-temperature superconductors are more
(15) temperature at which they become superconductors— promising than low-temperature superconductors
which in most instances is below –253°C (–423°F). for physicists hoping to make the benefits of
Cooling is normally accomplished by immersing the superconductors widely accessible.
elements in extremely expensive liquid helium. (C) The scientific benefits to science of
Then, in 1986, the discovery that a small number superconductors are just now being noticed
(20) of compounds demonstrated superconductivity at the by the general public due to a recently
relatively high temperature of –173°C (–279°F)—a invented device that utilizes superconducting
transition temperature reachable by immersion in technology.
relatively inexpensive liquid nitrogen—rejuvenated (D) Many technical fields, from medicine to
physicists’ hopes. But once again progress has been transportation, look forward to the benefits
(25) slower than expected, primarily because most of these of a recent breakthrough in the manufacture
so-called high-temperature superconductors belong to of superconductors.
the ceramics family, a group of highly fragile (E) While a few superconducting devices have
compounds. Nevertheless, one promising application been invented, they have a narrow range of
of high-temperature superconductors is their use in applications, so superconductivity’s potential
(30) superconducting quantum interference devices usefulness must be viewed skeptically.
(SQUIDs). Invented in the 1960s, SQUIDs are small
wirelike devices made of two superconductors 22. Which one of the following inferences concerning
separated by a layer of insulating material that can be superconductor technology is most supported by the
used to detect minuscule changes in electrical energy— information in the passage?
(35) changes roughly equivalent to the amount of energy
(A) Most materials can function as superconductors
required to move an electron 0.01 millimeter. This
capacity has allowed SQUIDs to be used in such tasks if they are cooled to appropriate temperatures.
as pinpointing the precise locations of brain lesions in (B) The prohibitive cost of using low-temperature
persons with focal epilepsy and identifying sources of superconductors is due in part to the difficulty
(40) geothermal energy beneath the earth’s surface. of maintaining them at or below their
Scientists have even used SQUIDs in attempts to transition temperatures.
detect the presence of a gravity wave transmitted (C) Superconductors can be manufactured via an
through space from an exploded supernova. expensive process available at only a few
But because they use ceramics, high-temperature facilities worldwide.
(45) SQUIDs suffer from the drawback of being more (D) The main obstacle to the development of
brittle and difficult to work with than are low- practical applications for superconductors is
temperature SQUIDs; for example, they cannot be the lack of superconducting materials that do
bent into shape as easily. One possibility being not become brittle at very cold temperatures.
explored for making high-temperature SQUIDs easier (E) Superconducting materials exist naturally in the
(50) to work with is to manufacture them as thin squares of earth and can be mined, but they are rare and
film rather than as wires. Another drawback is that extremely expensive to retrieve and refine.
SQUIDs manufactured with superconductors cooled
by liquid nitrogen are slightly less accurate than their
low-temperature counterparts. In certain contexts,
(55) such as sensitive medical procedures, low-temperature
SQUIDs would still be called for. Nevertheless,
because liquid nitrogen is inexpensive, slow to
GO ON TO THE NEXT PAGE.
evaporate, and readily available, high-temperature
superconductors remain the most promising route

Law School Admission Council


3 3
23. By stating that high-temperature SQUIDs are “slightly
3 3 -31-

24. The author mentions which one of the following as an


3
less accurate than their low-temperature counterparts” actual application of superconductors?
(lines 53–54), the author most likely intends to
(A) increasing train speeds
convey that
(B) transmitting geothermal energy
(A) the difference in performance between the two (C) moving electrons
types of SQUIDs is so small that one type (D) locating brain lesions
should not be favored over the other (E) streamlining computer systems
(B) the optimal operating temperature is more easily
maintained in low-temperature SQUIDs than in
high-temperature SQUIDs
(C) high-temperature SQUIDs are less sensitive to
changes in temperature than are
low-temperature SQUIDs
(D) high-temperature SQUIDs have no practical
applications for scientists because of this
deficiency
(E) high-temperature SQUIDs are not as sensitive to
changes in electrical charges as are
low-temperature SQUIDs

S T O P
IF YOU FINISH BEFORE TIME IS CALLED, YOU MAY CHECK YOUR WORK ON THIS SECTION ONLY.
DO NOT WORK ON ANY OTHER SECTION IN THE TEST.

Law School Admission Council


4 -32-
=
4 =
4 =
4 =
4
SECTION IV
4
Time—35 minutes
23 Questions

Directions: The questions in this section are based on the reasoning contained in brief statements or passages. For some
questions, more than one of the choices could conceivably answer the question. However, you are to choose the best answer; that
is, the response that most accurately and completely answers the question. You should not make assumptions that are by
commonsense standards implausible, superfluous, or incompatible with the passage. After you have chosen the best answer,
blacken the corresponding space on your answer sheet.

1. Biologist: The orb web is the most efficient form of Questions 3–4
spiderweb; its great efficiency is the reason that
All of the history books I have ever read treat Razia
so many different species of spiders independently
Sultan fairly. You say that this history book treats Razia
evolved this type of web.
Sultan fairly, and I assume that you are right. Therefore,
Entomologist: Actually the orb web is less efficient than this book must be one of the ones that I have read.
the dense and elaborate cobweb. The orb web, a
primitive form, is shared by many different 3. Which one of the following would, if true, allow the
species of spiders because it was inherited from conclusion to be properly drawn?
a common ancestor.
(A) The author has read some of the history books
The statements above commit the biologist and that do not treat Razia Sultan fairly.
entomologist to disagreeing about which one of (B) The author has read every history book that
the following? treats Razia Sultan fairly.
(A) The orb web is a primitive form of spiderweb. (C) The author no longer remembers what some of
(B) The cobweb is more elaborate than the orb web. the history books say about Razia Sultan.
(C) The orb web is the most common form of (D) Some history books do not treat Razia Sultan
spiderweb. fairly.
(D) The cobweb is the most common form of (E) The author has read only one history book that
spiderweb. treats Razia Sultan fairly.
(E) The orb web is the most efficient form of
spiderweb. 4. The flawed pattern of reasoning in the passage is
most closely paralleled in which one of the following
2. Anup: The development of a plain writing style in arguments?
seventeenth-century England was mainly due (A) Anyone who is good at playing chess is also
to an increase in the literacy rate. To reach good at solving puzzles. Gagan is not good at
moderately educated readers, writers simplified solving puzzles, so he must not be good at
their style. playing chess.
Kiran: No, the pivotal factor was the increasing interest (B) All ducks are web-footed. This bird must be
in science among the most highly educated web-footed because it is a duck.
people; a complex, artificial style, however (C) This house must be expensive because it is
beautiful, interfered with the presentation of made of brick, and inexpensive houses are
scientific facts. never made of brick.
(D) Either Chirag or Manisha must be wrong
Anup’s and Kiran’s comments indicate that they
because they chose different answers, and only
disagree about
one answer is correct.
(A) whether the quality of written works in (E) Those who disagree with me are my enemies.
seventeenth-century England increased or Rachana is my enemy. Therefore, it must be
decreased as a result of the development of a true that Rachana disagrees with me.
plain writing style
(B) the extent of the changes in writing style that
took place in seventeenth-century England
(C) whether there was an increase in the percentage
of people who were able to read in England
GO ON TO THE NEXT PAGE.
during the seventeenth century
(D) how widespread the dissemination of scientific
knowledge in seventeenth-century England was
(E) what was the primary cause of the development
of a plain writing style in seventeenth-century
England

Law School Admission Council


4 = = = =
4 4
5. Ten years ago, because of concern that vague laws
allowed people to shelter too much income in offshore
4 4
stories told by preliterate peoples are slightly
-33-

7. Scholar: Despite variations across cultures, all recorded


4
accounts, the Offshore Tax Shelter Limitation Act different versions of a small number of common
(OTSLA) was enacted to set clear limits on using story patterns. Although one might initially
offshore accounts to shield income from taxation. expect that this fact would make these stories
Paradoxically, however, the use of offshore tax shelters incapable of revealing much about the distinctive
has increased since OTSLA was enacted. features of the different cultures that have
produced them, I have found that they actually
Which one of the following, if true, most helps to
provide a rich source for understanding some of
resolve the apparent paradox described above?
the differences among these cultures.
(A) The tax authorities are charged with strictly
Which one of the following, if true, would explain the
enforcing OTSLA, and as a result many people
scholar’s unexpected finding?
have been prosecuted for violating it.
(B) The clear wording of OTSLA made it apparent (A) The fact that all stories from preliterate cultures
to more people that they could legally shelter are slightly different versions of a small
income in offshore accounts. number of common story patterns reveals that
(C) OTSLA is one of several laws enacted in the all of the different versions of the same story
last ten years that were intended to reduce have a common origin.
abuse of the system of taxation. (B) An understanding of differences among cultures
(D) Because of concern over the increasing use of can be gained only if the stories told in those
offshore tax shelters, legislators have recently cultures are based on dissimilar story patterns.
introduced a bill that would impose additional (C) The fact that written stories are not all
limits. variations of a small number of common story
(E) After OTSLA was enacted, many tax lawyers patterns reveals that there are vast differences
counseled their clients to avoid using offshore between preliterate and literate cultures.
tax shelters. (D) The ways in which different cultures choose to
subtly modify common story patterns reveal
6. Journalist: Contrary to popular opinion, it is more distinctive features of these different cultures.
dangerous for an individual to drive during the (E) The unrecorded stories of preliterate cultures
day than during the night. A recent study found would reveal significantly more about the
that in each of the last ten years, the number of differences among those cultures than the
traffic accidents that resulted in death was greater limited number of recorded stories do.
during the day than during the night.
Each of the following, if true, helps to strengthen the
argument EXCEPT:
(A) Only during the day are there more unsafe than
safe vehicles on the road. GO ON TO THE NEXT PAGE.
(B) There is decreased law enforcement presence
during the day.
(C) People drive more cautiously during the night
than during the day.
(D) The number of travelers per vehicle has
increased over the past ten years.
(E) People drive faster during the day than during
the night.

Law School Admission Council


4 -34-
=
4 =
4 =
4 =
4
8. Anthropologist: Bubonic plague typically does not
infect humans until after a large number of rats
10. In some patients, the course of heart disease has now
been successfully reversed without the use of prescription
4
have become infected. Historically, epidemics of drugs. Those patients participated in a medical experiment
bubonic plague have been associated with in which they followed a specific therapeutic regimen
enormous die-offs in rats. It has long been that consisted of a vegetarian diet, moderate exercise,
suspected that the Black Death, an epidemic that and daily meditation. Some observers concluded from
killed millions of people around the world in the this result that, to reverse the course of heart disease,
fourteenth century, was the same disease as prescription drugs are completely unnecessary for a
bubonic plague, but this is probably not right patient who adheres to this regimen.
since _______.
The observers’ conclusion is based on which one of
The conclusion of the anthropologist’s argument is the following assumptions?
most strongly supported if which one of the following
(A) Older people will benefit the most from this
completes the passage?
new approach because they are at high risk
(A) diseases other than the Black Death also killed from conventional drug and surgical approaches.
many people in the fourteenth century (B) Patients who follow the experiment’s
(B) there are no reports dating from the fourteenth therapeutic regimen but who do not refrain
century of large numbers of dead rats from smoking tobacco will benefit very little
(C) many diseases other than bubonic plague can be from the new treatment.
spread directly from one human to another (C) The therapeutic regimen followed in the
(D) widespread bubonic plague infection in rats experiment is not likely to be improved on in
does not always lead to an epidemic in the the treatment of heart disease.
human population (D) The therapeutic regimen followed in the
(E) bubonic plague is not the only disease that can experiment must be accompanied by the
be spread from rats to humans administration of cholesterol-reducing drugs
and medication to reduce blood pressure.
9. In an effort to deter crime, administrators at a university (E) There are no chronic heart disease patients
decided to improve the lighting in campus parking lots. whose condition has become progressively
Because of budgetary restrictions, only half of the worse despite a long-established life-style
university’s parking lots received improved lighting. just like the therapeutic regimen followed in
Unexpectedly, the number of complaints about poor the experiment.
lighting in university parking lots more than doubled
after this new lighting was installed.
Which one of the following most helps to explain the
unexpected result?
(A) Most people are not aware of any connection GO ON TO THE NEXT PAGE.
between improving outdoor lighting and
deterring crime.
(B) For the most part, students at the university
were unaware that the lighting in the parking
lots had been improved.
(C) Other than in parking lots, the university made
no improvements to its outdoor lighting.
(D) After the new lighting was installed, more
people used the lots with the improved lighting
and fewer people used the lots where the
lighting was not improved.
(E) Many people judged the lots with unimproved
lighting against the standard set by the lots
with improved lighting.

Law School Admission Council


4 = = = =
4 4
11. Volunteers watched a letter “X” move rapidly on a
computer screen. As they watched, the volunteers tried
4 4
13. Art historian: In 2004 several Buddhist religious
paintings dating between 800 A.D. and 900 A.D.
-35-
4
to record the movement of the “X” by pressing buttons were found in caves along the ancient trading
that corresponded to the regions through which the “X” route known as the Silk Road. After discovering
moved. The “X” moved seemingly at random but dried oils in samples from the paintings, some
actually in accordance with a complex set of 10 rules, researchers concluded that these paintings are the
about which the volunteers were not told. Because the oldest known instances of oil painting. However,
volunteers grew more adept at the task with practice, such a conclusion is unwarranted. In the Buddhist
they must have, in effect, determined through practice tradition travelers were encouraged to touch
some set of rules that helped them predict the way in religious paintings, and the oils could merely be
which the “X” would move. the residue of touching.
Which one of the following, if true, most helps to Which one of the following, if true, most seriously
strengthen the argument? weakens the art historian’s argument?
(A) When questioned afterward, the volunteers (A) Temperatures and humidity in the caves are
reported that they had found the task ideal for preserving oil-based paintings.
increasingly enjoyable as time elapsed. (B) Some of the oil-containing samples were from
(B) Some of the volunteers decided which buttons layers of paint that are several layers below
to press by watching other volunteers instead the surface.
of by watching the computer screen. (C) Some Buddhist religious paintings from well
(C) The volunteers’ performance declined sharply after 900 A.D. are known to have used oil paint.
when the rules governing the movement of the (D) Some contemporaneous paintings in other areas
“X” were later suspended and the “X” moved were not oil paintings but were coated with an
truly at random. oil-based varnish after the paint dried.
(D) The volunteers were chosen from a group (E) The paintings have been viewed and appreciated
of athletes who were highly skilled at by many non-Buddhists over the years.
coordinating the movement of their hands
with what they saw.
(E) When the volunteers were given a written list of
the 10 rules governing the movement of the
“X,” they found it difficult to calculate correctly
GO ON TO THE NEXT PAGE.
what the movement of the “X” would be from
any given starting point.

12. In the past twenty years, parents in North America


have been more frequently naming friends rather than
relatives as guardians of their children in case of the
parents’ deaths. These parents seem to place more
importance on parenting ability and geographical
location than on there being a family relationship
with potential guardians.
The situation described above most closely conforms to
which one of the following generalizations?
(A) The quality of family relationships has declined
in the last twenty years to the point where
people no longer depend on their relatives.
(B) Recently people have tended to value friends
more than family members.
(C) In recent years practical considerations have
become increasingly important in the choice
of guardians.
(D) Parents in the past were more concerned with
being good parents than with pleasing family
members.
(E) In recent years self-sufficiency has come to be
valued more than closeness among relatives.

Law School Admission Council


4 -36-
=
4 =
4 =
4 =
4
14. Situation: The trend toward treating certain illnesses
without surgery has improved the overall quality
15. Critic: Designers should be concerned about the
intended purpose of the artifacts that they create
4
of the lives of those suffering from these illnesses. and their ultimate effects on society for good or
However, the death rate among those people has ill. They should consider the utility of the
also increased. products they design and ensure that they satisfy
Analysis: Substantial improvements in the quality of the practical, everyday needs of the people for
life for members of a particular group may be whose use they design them. The artifacts in
accompanied by a greater chance of premature society are both a product and a reflection of
death for members of that group. specific economic, political, and cultural history.
They in turn help shape society and affect the
The analysis provided for the situation above would quality of people’s lives. Designers of artifacts
be most appropriate for which one of the following thus play a critical role, especially in a society as
situations? self-conscious about design as ours.
(A) Letting pet dogs roam free rather than keeping If the critic’s claims above are true and a designer is
them on leashes makes them happier, but these commissioned to design a new building, which one of
dogs run a greater risk of being killed by cars. the following can be properly concluded?
(B) Decreasing the use of antibiotics for routine
upper respiratory infections slows the (A) She should choose the design which satisfies
development of drug-resistant bacteria, the aesthetic tastes of the people who are
although more people may develop pneumonia. commissioning it.
(C) Feeding domestic cats large amounts of milk is (B) She should consider certain needs of the
not recommended by veterinarians because it prospective users of the building.
causes cats minor digestive tract problems, yet (C) She should ensure that her design will be
cats are fond of milk. acceptable to the current design establishment.
(D) Giving toys to laboratory mice reduces the stress (D) She should ensure that her building incorporates
of living in cages, although such measures do all the recent technological innovations in
not increase the life spans of the mice. building techniques.
(E) Irradiating fruits and vegetables can preserve (E) She should make the interests of the client
much food that otherwise would spoil, but the her paramount consideration when considering
nutritional value of such food is decreased. the design.

GO ON TO THE NEXT PAGE.

Law School Admission Council


4 = = = =
4 4
16. Darwin’s theory of evolution would never have been
conceived had it not been for the dynamic social and
4
Questions 17–18
4 -37-
4
Minister: MPs often receive large contributions for their
economic changes brought about by capitalism in
reelection campaigns from special-interest groups. In
nineteenth-century Britain. The nineteenth-century
return, MPs push through legislation that they would
British economy was also indispensable to Marx’s social
not otherwise support. We should therefore limit the
and economic theory. In other words, the capitalism of
amount of money that any one group is permitted to
nineteenth-century Britain was essential to the existence
contribute to a campaign in order to help ensure that
of both Marxism and Darwinism. Therefore without
any legislation that is enacted will benefit the country
Marxism, there would have been no Darwinism, and
as a whole, not just special-interest groups.
without Darwinism, there would have been no Marxism.
MP: But many special-interest groups work for the common
The reasoning in the argument is flawed because the good. By accepting their contributions and working on
argument legislation, we serve the common good more
(A) mistakes a historical explanation of a theory effectively than we would by wasting time on the extra
for a justification of that theory fund-raising that the contribution limits would require.
(B) fails to establish that new theories are determined
by social, economic, or historical forces 17. The main issue in dispute between the minister and
(C) infers the mutual interdependence of two things the MP is
from their each having the same precondition (A) whether MPs’ votes are influenced by
(D) treats two theories that both arose in a given campaign contributions
historical context as the only theories that (B) whether limiting campaign contributions is
could have arisen in that context likely to further the common good
(E) takes a relation of causal interdependence (C) how special-interest groups could best benefit
between two theories as an indication that the country as a whole
those theories express the same fundamental (D) how the minister can best raise funds for
principle reelection campaigns
(E) whether only special-interest groups that work
against the common good make large
campaign contributions

18. The MP’s first sentence counters the minister’s


argument by
(A) claiming that implementing the minister’s
proposal would have undesirable results
(B) casting doubt on the legitimacy of the minister’s
motives in advancing the argument
(C) introducing information challenging an unstated
assumption in the minister’s argument
(D) rejecting the premise that the minister states first
(E) arguing for a particular alternative to the
minister’s proposal

GO ON TO THE NEXT PAGE.

Law School Admission Council


4 -38-
=
4 =
4 =
4 =
4
19. At the southern end of Madagascar lie four wedge-
shaped land formations resulting from sediment deposits.
Questions 20–21
4
In 1854 the British scientist and explorer Richard Burton
Each covers more area than a large city and is as high
dismissed as superstition the view of certain native people
as a skyscraper. Since the deposits contain microfossils
in tropical areas that mosquito bites cause deadly fevers.
from the ocean floor and bits of natural glass consistent
Burton attributed this view to “the belief that mosquitoes
with the impact of a comet or asteroid, some geologists
and fevers become formidable about the same time.” Several
hypothesize that they were formed by a giant wave
decades later, medical researchers concluded that mosquito
resulting from the impact of an asteroid or comet in
bites are the principal means of transmission of two deadly
the Indian Ocean.
tropical diseases that have symptoms that include high fever.
Which one of the following, if true, most supports the
geologists’ hypothesis? 20. The information above can most reasonably serve
as part of an argument in support of which one of
(A) A wave would have had to be the size of a
the following?
50-story building to form the deposits.
(B) Similar deposits have been identified in other (A) Scientific dismissals of traditional, nonscientific
locations around the world, most of them explanations of the causes of various diseases
within several miles of an ocean. have historically turned out to be right more
(C) Large ocean waves that make landfall are often than they have turned out to be wrong.
sometimes caused by earthquakes, volcanos, (B) Hasty scientific judgments about hypotheses
or similar phenomena. based on certain observed data sometimes
(D) The microfossils date from about 5,000 years arise from prejudice concerning the source
ago, which is more recent than other estimates of such data.
of when the last asteroid or comet hit Earth. (C) Most scientists would agree that whenever
(E) There is a huge impact crater in the ocean floor two events regularly occur at about the same
in the middle of the Indian Ocean. time, one of these events must be a cause of
the other.
(D) Scientists seldom dismiss observed evidence
gathered by scientists of an earlier period.
(E) Beliefs held by members of a given cultural
community can be adequately assessed only
by people who are well acquainted with that
community’s language and customs.

21. Burton attributed the view of native people in tropical


areas about mosquitoes and deadly fevers to which one
of the following?
(A) failing to take into account that a condition was
present many times even without the presence
of the alleged causal factor
(B) making a judgment about an alleged effect
based on insufficient evidence about the
frequency of that alleged effect’s occurrence
(C) failing to distinguish genuine causes of a
condition from another factor that merely
coincided with that condition
(D) basing their belief on everyday occurrences
rather than on data collected in carefully
controlled experiments
(E) basing their belief about the cause of a
condition on something that is the result of
that condition

GO ON TO THE NEXT PAGE.

Law School Admission Council


4 = = = =
4 4
22. Recently a respected university investigated commercial
organizations that offered special courses for high-school
4 4
23. Municipal ordinance in the early 1990s: Smoking is
-39-

prohibited in bars in the city, but since the sole


4
students seeking to improve their grades, and found that purpose of this law is to protect employees from
students who had taken such courses received no higher secondhand smoke, there is one exception:
grades on average than students who had not done so. smoking is permitted in bars that are staffed
Obviously these courses did not in general help solely by their owners.
high-school students improve their grades. Application: Even though several bartenders and waiters
The reasoning in the argument is most vulnerable to work at Shalimar, a bar in the city, smoking is
criticism on the grounds that the argument permitted there.

(A) offers an ambiguous description of how two Which one of the following, if true, justifies the above
groups differ application of the ordinance?
(B) assumes without justification that a trend that (A) Most owners of bars located in the city are
has held in the past will continue to do so in concerned more about profit than about
the future secondhand smoke.
(C) fails to consider the differences that might have (B) Most owners of bars located in the city
existed between two groups before the factor are unaware of the exception to the
at issue was introduced smoking prohibition.
(D) uncritically accepts a claim without considering (C) At any given time, almost all of the patrons at
the qualifications of the individual or group Shalimar are regular smokers.
making the claim (D) Everyone who works at Shalimar is a regular
(E) identifies one thing as the cause of another smoker.
when the evidence given indicates that the (E) Everyone who works at Shalimar is an owner
second is the cause of the first of the business.

S T O P
IF YOU FINISH BEFORE TIME IS CALLED, YOU MAY CHECK YOUR WORK ON THIS SECTION ONLY.
DO NOT WORK ON ANY OTHER SECTION IN THE TEST.

Please remain seated until all test books and answer sheets have been collected and checked.

Law School Admission Council


-40-

NO TEST MATERIAL ON THIS PAGE.

Law School Admission Council


Acknowledgment is made to the following sources from which material has been adapted for use in this test booklet:

Sandra Blakeslee, “Ancient Crash, Epic Wave.” ©November 14, 2006 by The New York Times.

John Clarke, “SQUIDs.” ©1994 by Scientific American, Inc.

Suzanne Lebsock, “They Helped Themselves.” ©1993 by The New York Times.

Michael Rosenthal, “The Problem of Fiction.” ©1985 by Morris Beja.

“Shaking Up the Day.” ©1988 by Discover, Inc.

41

Law School Admission Council


ANSWER KEY

SECTION I

1. D 8. E 15. C
2. A 9. C 16. A
3. E 10. E 17. C
4. B 11. E 18. B
5. C 12. D 19. C
6. D 13. B 20. D
7. B 14. D

SECTION II

1. A 8. B 15. A 22. C
2. B 9. E 16. A 23. A
3. B 10. C 17. D 24. A
4. E 11. D 18. C 25. B
5. B 12. C 19. B
6. B 13. E 20. C
7. A 14. D 21. D

SECTION III

1. A 8. C 15. A 22. B
2. D 9. A 16. E 23. E
3. E 10. D 17. C 24. D
4. B 11. B 18. E
5. A 12. A 19. A
6. D 13. C 20. D
7. B 14. B 21. B

SECTION IV

1. E 8. B 15. B 22. C
2. E 9. E 16. C 23. E
3. B 10. E 17. B
4. E 11. C 18. C
5. B 12. C 19. E
6. D 13. B 20. B
7. D 14. A 21. C

42

Law School Admission Council


Law School Admission Council

Вам также может понравиться